Calculating Newton's Method: Step-by-Step Guide

Click For Summary
The discussion focuses on applying Newton's method to find a stationary value of a piecewise function defined for different intervals. The initial formula for Newton's method is correctly stated as x_{n+1}=x_{n}-f(x_{n})/f'(x_{n}). The user is advised to evaluate cos(0) and sin(0) to proceed with calculating the first iterate using x_{0}=1. The conversation emphasizes the importance of substituting these values accurately to continue the iterative process. The next steps involve computing the second iterate to approximate the stationary value c within the specified range.
jisbon
Messages
475
Reaction score
30
Homework Statement
Let $$f(x) =
\begin{cases} \dfrac{x^3-1}{\sqrt{x}-1}, & x > 1\\
cos(x-1)-x^2, & x \leq 1\end{cases}$$

Use Newton's method with ##x_{0} =1##, compute the second iterate to approximate value ##c## where ##c## is a stationary value that lies in the x-axis for some ##0<c<1##
Relevant Equations
-
Since the Newton's method is as follows:

$$x_{n+1}=x_{n}-\frac{f(x_{n})}{f'(x_{n})}$$

$$x_{1}=x_{0}-\frac{cos(0)-1}{-sin(0)-2}$$

Is this correct? What should I proceed on from here?
 
Last edited by a moderator:
Physics news on Phys.org
jisbon said:
Homework Statement: Let $$f(x) =
\begin{cases} \dfrac{x^3-1}{\sqrt{x}-1}, & x > 1\\
cos(x-1)-x^2, & x \leq 1\end{cases}$$

Use Newton's method with ##x_{0} =1##, compute the second iterate to approximate value ##c## where ##c## is a stationary value that lies in the x-axis for some ##0<c<1##
Homework Equations: -

Since the Newton's method is as follows:

$$x_{n+1}=x_{n}-\frac{f(x_{n})}{f'(x_{n})}$$

$$x_{1}=x_{0}-\frac{cos(0)-1}{-sin(0)-2}$$

Is this correct?
Yes, as far as you went
jisbon said:
What should I proceed on from here?
Evaluate cos(0) and sin(0) and substitute in the value for ##x_0##.
 
Question: A clock's minute hand has length 4 and its hour hand has length 3. What is the distance between the tips at the moment when it is increasing most rapidly?(Putnam Exam Question) Answer: Making assumption that both the hands moves at constant angular velocities, the answer is ## \sqrt{7} .## But don't you think this assumption is somewhat doubtful and wrong?

Similar threads

Replies
6
Views
2K
Replies
2
Views
2K
Replies
4
Views
2K
  • · Replies 3 ·
Replies
3
Views
1K
  • · Replies 2 ·
Replies
2
Views
588
  • · Replies 2 ·
Replies
2
Views
1K
Replies
1
Views
1K
  • · Replies 105 ·
4
Replies
105
Views
6K
  • · Replies 1 ·
Replies
1
Views
2K
  • · Replies 1 ·
Replies
1
Views
1K